LSAT and Law School Admissions Forum

Get expert LSAT preparation and law school admissions advice from PowerScore Test Preparation.

 Administrator
PowerScore Staff
  • PowerScore Staff
  • Posts: 8917
  • Joined: Feb 02, 2011
|
#37053
Please post below with any questions!
 harvoolio
  • Posts: 63
  • Joined: Apr 25, 2018
|
#46189
I chose (D) over (B) because the stimulus did not state this.

I also felt "inevitably" was strong in (D) but the stimulus never uses any qualifier so from the stimulus' perspective "inevitably" has been established.

In retrospect, I think (D) is wrong and (B) is right because (B) matches the degree nature of the verb "diverts" in the stimulus whereas (D) is binary. (D) uses the word "lose" which is binary and means your thoughts are gone whereas the stimulus said "diverts" which means reduce. (B) uses a synonym of "diverts" in "detracts" which also means to reduce.

Is this logic correct? Thanks.
 Alex Bodaken
PowerScore Staff
  • PowerScore Staff
  • Posts: 136
  • Joined: Feb 21, 2018
|
#46277
harvoolio,

Thanks for the question! Yes, your logic is sound - the issue with answer choice (D) is the phrase "inevitably lose," which connotes that a binary action will happen every time...the stimulus doesn't support this. Rather, the stimulus suggests that the listener's attention will be distracted - and detracting from driving performance falls under the umbrella of actions that can happen when one's attention is distracted. This makes (B) the correct answer choice.

Hope that helps!
Alex
User avatar
 andrewb22
  • Posts: 18
  • Joined: May 04, 2021
|
#87079
I got this question correct, but the approach to MBT questions really had me over thinking this. If we are only applying what's in the stimulus to what a correct answer can be, (B) is a leap and requires an assumption. In order for (B) to be correct you have to assume that diverting one's attention leads to a detraction of driver performance. I know the whole idea with MBT questions is to make an inference based off info in the stimulus, but to me you have to bring in information outside of the stimulus: a lack of attention will lead to worse driving ability. While I think this is true in the real world, I don't understand how you can make that leap from the stimulus alone. There is nothing in the stimulus that indicates a lack of attention will lead to a decline in ability at anything, just that you are not completely focused. Does this fall under the "most strongly support"/"plausible assumption based on common sense standards" category?

Ultimately, I chose (B) because it was the best answer(and many questions have answers that aren't great), but I'm curious if there is something wrong with my process here or if I'm overthinking it.
 Robert Carroll
PowerScore Staff
  • PowerScore Staff
  • Posts: 1787
  • Joined: Dec 06, 2013
|
#87091
Andrew,

This is a Most Strongly Supported variety of Must Be True, so some very modest assumptions are acceptable, and I think the assumption, if any, you have to make to prove answer choice (B) is very modest. As you point out, the assumption would be that a driver with at least some of their attention diverted must perform at least somewhat worse at driving. That should be enough for the answer. Even so, I don't know if I'd classify that as an assumption anyway. Consider - the stimulus says that the one-sided cell-phone conversation "diverts listeners’ attention from" their activity. The "diversion" language indicates to me that the attention can't be as strong when it's diverted - some of the attention goes from driving to the other activity. So the amount of attention devoted to driving is diminished. Less attention should mean some effect on performance.

Robert Carroll

Get the most out of your LSAT Prep Plus subscription.

Analyze and track your performance with our Testing and Analytics Package.